Difficulté sur deux suites

Un problème, une question, un nouveau théorème ?

Messages : 3

Inscription : 22 mars 2020 15:08

Profil de l'utilisateur : Élève de lycée

Difficulté sur deux suites

Message par Mourien » 22 mars 2020 15:17

Bonjour,

Je bloque sur l'exercice suivant:

$ \forall n,u_{n+2}\leq\dfrac{u_n+u_{n+1}}2 $

Avec $ (un) $ définie positive. Il s'agit de montrer qu'elle converge.

En considérant la suite $ max(u_n, u_{n+1}) $ qui converge, peut on espérer aboutir ?


Et sinon, j'aimerai aussi résoudre

$ \forall a, b, u_{a+b} \leq u_a+u_b $

Cette fois il s'agit de monter que $ v_n=\dfrac{u_n} n $ 'converge' vers sa borne inférieure (éventuellement$ -\infty $).

Tout ce que j'ai c'est que $ u_n\leq n u_1 $ et j'essaye vainement de montrer $ v_n $ décroissante...

Merci d'avance!
PCSI ; MP* ; ENS de Lyon

Messages : 3901

Inscription : 04 sept. 2005 19:27

Profil de l'utilisateur : Élève de lycée

Re: Difficulté sur deux suites

Message par JeanN » 22 mars 2020 16:22

Exo1 :
Oui, ça va bien fonctionner.
Tu peux par exemple démontrer par retour à la définition que (u_n) converge vers la même limite que la limite de la suite $max(u_n, u_{n+1})$

Exo2 :
Retour à la définition nécessaire également. On note i la borne inf.
Tu fixe eps>0, tu choisis N tel que $u_N/N<i+eps/2$
Ensuite, tu prends $n>N$, tu fais la division euclidienne de n par N et tu cherches à majorer par $i+eps/2$+ une suite qui tend vers 0 quand n tend vers l'infini.
Il ne te restera plus qu'à conclure à la manière de Césaro.
Professeur de maths MP Lycée Sainte-Geneviève

Messages : 3

Inscription : 22 mars 2020 15:08

Profil de l'utilisateur : Élève de lycée

Re: Difficulté sur deux suites

Message par Mourien » 22 mars 2020 17:36

D'abord merci !

Donc pour l'exo 1 je fixe $ \epsilon>0 $. A partir de $ N, max(u_n, u_{n+1}) \in[l-\epsilon,l+\epsilon] $.

Supposons il existe $n>N, u_n \not \in [l-\epsilon, l+\epsilon] $

On a donc $u_n<l-\epsilon$ or $u_{n-1}<l+\epsilon$ donc $u_{n+1}<l$ ce qui est absurde car $l$ ne majore ni $u_n$ ni $u_{n+1}$ donc pas leur max alors que la suite décroît donc est minorée par $l$.

Edit ; j'ai corrigé.

Pour l'exo 2

Du coup en notant $N<n=qN+r, r<N$

On a $\dfrac{u_n} n\leq \dfrac{q u_N}n+\dfrac{u_r} n\leq\dfrac{u_N}N\dfrac{qN} {qN+r} +\dfrac{u_r}n$

Or $u_r$ borné donc en divisant par $n$ ça tend vers $0$. Et $\dfrac {qN} {qN+r} $ tend vers $1$

Donc $\dfrac{u_n} n\leq i+3\epsilon$ a partir d'un certain rang.
Dernière modification par Mourien le 22 mars 2020 18:16, modifié 1 fois.
PCSI ; MP* ; ENS de Lyon

Messages : 3901

Inscription : 04 sept. 2005 19:27

Profil de l'utilisateur : Élève de lycée

Re: Difficulté sur deux suites

Message par JeanN » 22 mars 2020 18:05

ok pour le premier exo
Pour le deuxième, il y a un petit bug dans la deuxième majoration (celle qui fait apparaitre i et eps)
Professeur de maths MP Lycée Sainte-Geneviève

Messages : 0

Inscription : 25 janv. 2020 17:53

Profil de l'utilisateur : Élève de lycée

Re: Difficulté sur deux suites

Message par ROH2F(x) » 23 mars 2020 17:15

Pour le premier exo tu supposes que le max converge ou tu le montres ?

Messages : 3901

Inscription : 04 sept. 2005 19:27

Profil de l'utilisateur : Élève de lycée

Re: Difficulté sur deux suites

Message par JeanN » 23 mars 2020 17:21

Il le montre en utilisant le théorème de limite monotone.
Professeur de maths MP Lycée Sainte-Geneviève

Messages : 0

Inscription : 25 janv. 2020 17:53

Profil de l'utilisateur : Élève de lycée

Re: Difficulté sur deux suites

Message par ROH2F(x) » 23 mars 2020 18:30

Ah oui je vois comment le prouver maintenant merci.

Répondre